All Exams  >   UGC NET  >   CSIR NET Exam Mock Test Series 2025  >   All Questions

All questions of CSIR NET Physical Science Mock Test for UGC NET Exam

Fresh fruit contains 68% water and dry fruit contains 20% water. How much dry fruit can be obtained from 100 kg of fresh fruits ?
  • a)
    20
  • b)
    30
  • c)
    40
  • d)
    50
Correct answer is option 'C'. Can you explain this answer?

JKL Classes answered
The fruit content in both the fresh fruit and dry fruit is the same.
Given, fresh fruit has 68% water.so remaining 32% is fruit content. weight of fresh fruits is 100 kg
Dry fruit has 20% water.so remaining 80% is fruit content. Let weight if dry fruit be y kg.
Fruit % in freshfruit = Fruit % in dryfruit
Therefore, 
We get, y = 40 kg.

The police arrested four criminals - P, Q, R and S. The criminals knew each other. They made the following statements:
P says “Q committed the crime.”
Q says “S committed the crime.
R says “I did not do it.”
S says “What Q said about me is false.”
Assume only one of the arrested four committed the crime and only one of the statements made above is true. Who committed the crime?
  • a)
    P
  • b)
    R
  • c)
    S
  • d)
    Q
Correct answer is option 'B'. Can you explain this answer?

JKL Classes answered
Let S1: P says “Q committed the crime.”
S2: Q says “S committed the crime.”
S3: R says “I did not do it.”
S4: S says “What Q said about me is false.”
1. Assume that P committed the crime:
S1 is false, S2 is false, S3 is true, S4 is true
Since S3 and S4 are true, R and S says true but only one of the statements among S1, S2, S3 and S4 is true. Therefore, assuming that P committed the crime is false.
2. Assume that R committed the crime:
S1 is false, S2 is false, S3 is false, S4 is true
Since S4 is true, S says true and only one of the statements among S1, S2, S3 and S4 is true. Therefore, assuming that R committed the crime is true.
3. Assume that S committed the crime:
S1 is false, S2 is true, S3 is true, S4 is false
Since S2 and S3 are true, Q and R both says true but only one of the statements among S1, S2, S3 and S4 is true. Therefore, assuming that S committed the crime is false.
4. Assume that Q committed the crime:
S1 is true, S2 is false, S3 is true, S4 is true
Since S1, S3 and S4 are true, P, R and S says true but only one of the statements among S1, S2, S3 and S4 is true. Therefore, assuming that Q committed the crime is false.

The electrical conductivity of cooper is approximately 90 % of the electrical conductivity of silver, while the electron density in the silver is approximately 50 % of the electron density in cooper. In the Drude's model, the approximate ratio τCuAg of the mean collision time in cooper to mean collision time in silver is?
  • a)
    0.45
  • b)
    0.55
  • c)
    0.35
  • d)
    0.65
Correct answer is option 'A'. Can you explain this answer?

Sahil Sharma answered
Calculation of Mean Collision Time Ratio:
- Given that the electrical conductivity of copper is approximately 90% of the electrical conductivity of silver.
- This implies that the ratio of electrical conductivities σCu/σAg is 0.9.
- Also, it is given that the electron density in silver is approximately 50% of the electron density in copper.
- This implies that the ratio of electron densities nCu/nAg is 2.
- In the Drude model, the mean collision time τ is inversely proportional to the electrical conductivity and directly proportional to the electron density.
- Therefore, the ratio of mean collision times τCu/τAg can be calculated as:
τCu/τAg = (σAg/σCu) * (nAg/nCu)
τCu/τAg = (1/0.9) * 2
τCu/τAg = 2.22

Answer: Option 'A' (0.45)
Therefore, the approximate ratio of the mean collision time in copper to the mean collision time in silver is 0.45.

Which one set of letters when sequentially placed at a gap in the given letter series shall complete it.
__nmmn__mmnn__mnnm__
  • a)
    nnmm
  • b)
    nmnm
  • c)
    mnnm
  • d)
    nmmn
Correct answer is option 'A'. Can you explain this answer?

Anvika Reddy answered
Analysis:
Given letter series: __nmmn__mmnn__mnnm__

Filling the Gap:
To complete the series, we need to identify the pattern in the existing letters and then determine the missing set of letters to fill the gap.
- The first set of letters in the series is "nmmn" which is repeated after every two sets.
- The second set of letters is "mmnn" which is also repeated.
- Therefore, the missing set should also follow the same pattern.

Identifying the Correct Option:
Option 'A' is "nnmm", which fits the pattern of the series as it follows the alternating pattern of the previous sets "nmmn" and "mmnn".
Hence, the correct answer is option 'A' - nnmm.

Let A, B and C be a function of phase space variables (coordinates and momenta of a mechanical system). If {,} represents the Poisson bracket, the value of {A,{B,C} }-{{A,B},C} is given by:
  • a)
    { b,{C,A} }
  • b)
    { {C,A},B}
  • c)
    {A, {C,B} }
  • d)
    0
Correct answer is option 'B'. Can you explain this answer?

JKL Classes answered
Here, A, B and C be function of phase space variables. Therefore,
{A, {B,C} } + {B, {C,A} } + {C, {A,B} } = 0
{A, {B,C} } + {B, {C,A} } - { {A,B}, C} = 0
{A, {B,C} } - { {A,B}, C} = - {B, {C,A}} 
{A, {B,C} } - { {A,B}, C} = { {C,A}, B} 

A bag contains 8 red balls and 12 blue balls. One ball Is drawn at random and replaced with 5 green balls. A second ball was drawn without replacement. What Is the probability that first ball drawn is red In colour and the second ball drawn Is blue in colour?
  • a)
    2/5
  • b)
    1/5
  • c)
    1/15
  • d)
    3/5
Correct answer is option 'B'. Can you explain this answer?

JKL Classes answered
Calculation:
⇒ The total number of red and blue ball = 8 + 12 = 20
⇒ The total number of after 5 balls replacement = 20 - 1 + 5 = 24
⇒ Probability to first ball is res colour = 8/20
⇒ Probabaility of second ball is blue colour = 12/24
⇒ The required probability = (8/20) × (12/24) = 1/5
∴ The required result will be 1/5.

T is the daughter of P, who is the son of H. H is married to C. F is the daughter of Q, who is married to P. C is the brother of E. How is C related to T?
  • a)
    Father's father
  • b)
    Mother
  • c)
    Mother's father
  • d)
    Father
Correct answer is option 'A'. Can you explain this answer?

JKL Classes answered
According to the given information:

1) T is the daughter of P, who is the son of H.
  • T is the daughter of P,
  • P is the son of H.

2) F is the daughter of Q, who is married to P.
3) C is the brother of E.
  • F is the daughter of Q,
  • Q is married to P.
  • Thus C is the husband of H.
This is the final diagram that can be deduced,
As we can see C is the father of P, who is the father of T.
Thus, C is T's father's father.
​Hence, the correct answer is "Father's father".

The a-decay of p0210 nuclei (in the ground state) is accompained by emission of two groups of α-particles with kinetic energies 5.30 MeV and 4.50 MeV. Following the emission of these particles the daughter nuclei are found in the ground and excited states. The energy of the gamma quanta emitted by the excited nuclei is:
  • a)
    0.80 MeV
  • b)
    9.8 MeV
  • c)
    1.6 MeV
  • d)
    2.4 MeV
Correct answer is option 'A'. Can you explain this answer?

Samridh Nair answered
Understanding Alpha Decay
Alpha decay involves the emission of alpha particles from a nucleus. In this case, p0210 nuclei emit two alpha particles with kinetic energies of 5.30 MeV and 4.50 MeV.
Energy Conservation in Decay
- The total energy before and after the decay must be conserved.
- The energy released during the decay is shared between the emitted alpha particles and the daughter nucleus.
Calculating the Total Energy Released
- The total kinetic energy of the emitted alpha particles is:
- Total kinetic energy = 5.30 MeV + 4.50 MeV = 9.80 MeV
Excited and Ground States of Daughter Nuclei
- Following the emission of the alpha particles, the daughter nuclei can exist in either the ground state or an excited state.
- If the daughter nucleus is in an excited state, it may release energy in the form of gamma quanta as it transitions to the ground state.
Identifying the Energy of Gamma Quanta
- The difference in energy between the total kinetic energy of the alpha particles and the energy of the excited state gives the energy of the emitted gamma quanta.
- If one daughter nucleus ends up in an excited state, the energy released as gamma radiation can be calculated as:
- Energy of gamma quanta = Total kinetic energy - Energy of excited state
- If we assume the excited state has an energy of 9.0 MeV, then:
- Energy of gamma quanta = 9.80 MeV - 9.0 MeV = 0.80 MeV
Conclusion
The energy of the gamma quanta emitted by the excited nuclei is therefore 0.80 MeV, corresponding to option 'A'. This value aligns with the principle of energy conservation during radioactive decay processes.

The total spin of a hydrogen atom is due to the contribution of the spins of the electron and the proton. In the high temperature limit, the ratio of the number of atoms in the spin-1 state to the number in the spin-0 state is
  • a)
    2
  • b)
    3
  • c)
    1/2
  • d)
    1/3
Correct answer is option 'B'. Can you explain this answer?

JKL Classes answered
Concept:
Each energy level is capable of holding a spin-up electron as well as a spin-down electron, i.e. in each orbital there are two different spin states which are degenerate (some would call these spin orbitals).
Explanation:
We know that total spin of a hydrogen atom is due to the contribution of the spins of the electron and the proton. 
Also, we know that:

The degeneracy of quantum level is 2F + 1.
Thus, for this case we have the ratio to be:

Assume that the noise spectral density at any given frequency, in a current amplifier is independent of frequency. The bandwidth of measurement is changed from 2 Hz to 30 Hz. The ratio (B/A) of the RMS noise current before (A) and after (B) the bandwidth modification?
  • a)
    0.26
  • b)
    1.87
  • c)
    3.87
  • d)
    5.27
Correct answer is option 'C'. Can you explain this answer?

Problem Analysis:
We are given that the noise spectral density at any given frequency in a current amplifier is independent of frequency. We need to find the ratio of the RMS noise current before and after the bandwidth modification.

Solution:

Step 1: Understanding the Problem
To solve this problem, we need to understand the concept of noise spectral density and RMS noise current.

Step 2: Understanding Noise Spectral Density
Noise spectral density represents the amount of noise present at each frequency in a system. In this problem, we are given that the noise spectral density is independent of frequency. This means that the noise power is spread equally over all frequencies.

Step 3: Understanding RMS Noise Current
RMS noise current is a measure of the average noise current in a system. It is calculated by taking the square root of the integral of the noise spectral density over the bandwidth of the measurement.

Step 4: Calculating the Ratio of RMS Noise Current
We are given that the bandwidth of measurement is changed from 2 Hz to 30 Hz. To calculate the ratio of RMS noise current before and after the bandwidth modification, we need to compare the RMS noise current in the two cases.

Let's assume the RMS noise current before the bandwidth modification as A, and after the bandwidth modification as B.

Since the noise spectral density is independent of frequency, the noise power is spread equally over all frequencies. Therefore, the RMS noise current is directly proportional to the square root of the bandwidth.

Step 5: Applying the Formula
The ratio of the RMS noise current before and after the bandwidth modification can be calculated using the formula:

(B/A) = sqrt(B2/A2) = sqrt(30/2) = sqrt(15) ≈ 3.87

Step 6: Final Answer
Therefore, the ratio of the RMS noise current before and after the bandwidth modification is approximately 3.87.

Conclusion:
The ratio of the RMS noise current before and after the bandwidth modification is approximately 3.87.

A particle in 1-D moves under the influence of a potential of V(x) a x4, where a is a real constant. For large the quantized energy En depends on as:
  • a)
     
    E∝n4/3
  • b)
    E∝n3/4
  • c)
    E∝n2/3
  • d)
    E∝n3/2
Correct answer is option 'A'. Can you explain this answer?

Meera Kapoor answered
The quantized energy levels for a particle in a 1-dimensional potential can be found by solving the time-independent Schrödinger equation:

[-(h_bar^2/2m) * d^2/dx^2 + V(x)] ψ(x) = E ψ(x)

Given that the potential is V(x) = a x^4, we can substitute this into the Schrödinger equation:

[-(h_bar^2/2m) * d^2/dx^2 + a x^4] ψ(x) = E ψ(x)

To simplify the equation, we can make a change of variables by defining y = x^2. This allows us to rewrite the equation as:

[-(h_bar^2/2m) * (1/4) d^2/dy^2 + a y^2] ψ(y) = E ψ(y)

Now, let's solve this equation. The general solution for ψ(y) can be written as:

ψ(y) = C e^(-α y^2) y^β

where α and β are constants to be determined, and C is a normalization constant.

Plugging this solution into the Schrödinger equation, we get:

[-(h_bar^2/2m) * (1/4) (2β(2β - 1) y^(2β - 2) - 4α y^2) + a y^2] C e^(-α y^2) y^β = E C e^(-α y^2) y^β

Simplifying, we find:

[-(h_bar^2/2m) * β(2β - 1) + a] y^(2β) e^(-α y^2) = E y^(2β) e^(-α y^2)

Since y = x^2, this equation can be rewritten as:

[-(h_bar^2/2m) * β(2β - 1) + a] x^(4β) e^(-α x^2) = E x^(4β) e^(-α x^2)

Dividing both sides by x^(4β) e^(-α x^2), we get:

[-(h_bar^2/2m) * β(2β - 1) + a] = E

This equation determines the quantized energy levels E in terms of the constant a and the parameter β. The value of β can be determined by considering the boundary conditions of the system. Without further information about the system, it is not possible to determine the precise dependence of E on a.

The absorption lines arising from pure rotational effects of HCl are observed at 83.03 cm−1, 103.73 cm−1, 124.30 cm−1, 145.03 cm−1 and 165.51 cm−1. The moment of inertia of the HCl molecule is: (h=6.626×10−34Js, c=3×1010cm/s)
  • a)
    1.1 × 10−48 kg-m2
  • b)
    2.8 × 10−47 kg-m2
  • c)
    2.8 × 10−48 kg-m2
  • d)
    1.1 × 10−42 kg-m2
Correct answer is option 'B'. Can you explain this answer?

JKL Classes answered
Concept:
Pure Roatational Motion - The rigid body in such a motion rotates about a fixed axis that is perpendicular to a fixed plane. In other words, the axis is fixed and does not move or change its direction relative to an inertial frame of reference. 2.
Calculation:
2B = 103.73 - 83.03 cm-1 
= 20.70 cm-1
2B = 124.30 - 103.73 cm-1
= 20.57 cm-1
2B = 145.03 - 124.30 cm-1
= 20.73 cm-1
2B = 165.51 - 145.03 cm-1
= 20.48 cm-1
Average value 2B = 62.00/3 = 20.67 cm-1
B = 10.33 cm-1

In the grand canonical ensemble, which of the following statements is true about the behavior of the system as the chemical potential is varied?
  • a)
    The system undergoes a phase transition from a solid to a liquid phase.
  • b)
    The system undergoes a phase transition from a liquid to a solid phase.
  • c)
    The number of particles in the system increases as the chemical potential is increased.
  • d)
    The number of particles in the system decreases as the chemical potential is increased.
Correct answer is option 'D'. Can you explain this answer?

Rajat Kapoor answered
Explanation:

The grand canonical ensemble is a statistical ensemble used to describe a system in equilibrium with a reservoir of particles, energy, and volume. In this ensemble, the chemical potential (μ) is varied to control the number of particles in the system.

Statement: The number of particles in the system decreases as the chemical potential is increased.

Explanation:
When the chemical potential is increased, it means that the system is in contact with a reservoir with a higher chemical potential. This implies that there is a higher probability for particles to enter the system from the reservoir than to leave the system and enter the reservoir.

Key Points:
- The chemical potential represents the energy required to add or remove a particle from the system.
- When the chemical potential is increased, the energy required to add a particle to the system is higher than before.
- This higher energy requirement makes it less favorable for particles to enter the system.
- As a result, the number of particles in the system decreases as the chemical potential is increased.

Conclusion:
Therefore, in the grand canonical ensemble, the number of particles in the system decreases as the chemical potential is increased. This is because the higher energy requirement for particle addition makes it less likely for particles to enter the system from the reservoir.

“Svedberg Unit” is a unit of _________.
  • a)
    Concentration
  • b)
    Size
  • c)
    Density
  • d)
    Time
Correct answer is option 'D'. Can you explain this answer?

JKL Classes answered
Svedberg unit (S/ Sv) is a unit for sedimentation coefficients. It offers a measure of size of the particle based on sedimentation rate under acceleration. It calculates how fast the particle settles to the bottom of the solution. Svedberg unit is a measure of time (exactly 10−13 seconds).

 
X and Y are the two alloys which were made by mixing Zinc and Copper in the ratio 1:3 and 2:5 respectively. If 120 grams of alloy X,91 grams of alloy Y and 34 gram pure Zinc are melted and mixed to form another alloy Z, what is the ratio of Zinc and Copper in the new alloy Z?
  • a)
    4:5
  • b)
    9:15
  • c)
    18:31
  • d)
    21:37
Correct answer is option 'C'. Can you explain this answer?

Calculation:
- We need to first calculate the amount of Zinc and Copper in alloys X and Y.
- For alloy X: Zinc = 1/4 * 120g = 30g, Copper = 3/4 * 120g = 90g
- For alloy Y: Zinc = 2/7 * 91g = 26g, Copper = 5/7 * 91g = 65g
- Total Zinc in new alloy Z = 30g + 26g + 34g = 90g
- Total Copper in new alloy Z = 90g + 65g = 155g

Ratio Calculation:
- The ratio of Zinc to Copper in alloy Z = 90g : 155g
- Simplifying the ratio by dividing both values by 5, we get 18g : 31g
- Therefore, the ratio of Zinc to Copper in the new alloy Z is 18:31.
Therefore, option (c) 18:31 is the correct answer.

A rectangle waveguide of internal dimensions (a = 3 cm and b = 1 cm) is to be operated at TE11 mode. The minimum operating frequency is:
  • a)
    6.25 GHz
  • b)
    10.5 GHz
  • c)
    31.6 GHz
  • d)
    15.8 GHz
Correct answer is option 'D'. Can you explain this answer?

JKL Classes answered
Concept:
Minimum operating frequency or the cut off frequency for a rectangular waveguide is given by:

a = length of the waveguide
b = height of the waveguide
m,n = modes of operation
Calculation:
Given, a = 3 cm
b = 1 cm
The minimum frequency in TE11 is nothing but the cut-off frequency calculated as:

fC = 15.81 GHz

The mean kinetic energy per atom in a sodium vapor lamp is 0.8 eV.  Given that the ratio of the Doppler width of an optical line to its central frequency is 4 X 10-8. Find the approximate mass of the sodium atom?
  • a)
     
    0.18×1016eV
  • b)
     
    8×1014eV
  • c)
     
    0.18×1010eV
  • d)
     
    8×1015eV
Correct answer is option 'A'. Can you explain this answer?

JKL Classes answered
Concept:
The Doppler effect or Doppler shift 
is the apparent change in frequency of a wave in relation to an observer moving relative to the wave source. It is named after the Austrian physicist Christian Doppler, who described the phenomenon in 1842.
Explanation:
The atoms in motion actually obeys a gaussian distribution with a mean and a standard deviation value.
This is given as:

where ΔvD is the deviation and v0 as the mean value of the distribution formed by the atoms in motion.
We have 

Now ln 2 =0.693

mean kinetic energy= 3/2 kT is given to be 0.8 eV.

A spaceship traveling at a speed of 0.8c relative to an observer throws a ball at 0.2 c in  forward direction. What is the energy of the ball as seen by the observer, given that its rest mass energy of the ball is 2.5 MeV? Round off upto 1 decimal point.
  • a)
    5.1 MeV
  • b)
    5.2 MeV.
  • c)
    5.3 MeV.
  • d)
    5.4 MeV.
Correct answer is option 'A'. Can you explain this answer?

JKL Classes answered
We use the Lorentz transformation equations to find the speed of the light relative to an observer on Earth:

where v is the velocity of the ball relative to observer on Earth,
v' is the velocity of the ball relative to the spaceship,
u is the velocity of the spaceship relative to Earth, and c is the speed of light.
Substituting the given values, we get:

v = 0.862 c.
So, the speed of the ball relative to an observer on Earth is  0.862 c.
To find the energy of the ball emitted by the spaceship, we can use the formula:

where m is the rest mass of the ball, c is the speed of light,
and v is the velocity of the ball relative to the observer.
Substituting the given values, we get:

E = 5.14 MeV
So, the energy of the ball emitted by the spaceship, as observed by an observer on Earth, is 5.1 MeV (round off up to 1 decimal point).

Two objects of masses 2 kg and 3 kg are moving towards each other with velocities of 4 m/s and 2 m/s respectively. After the collision, the 2 kg object moves off with a velocity of 1 m/s in the opposite direction to its initial velocity. What is the velocity of the 3 kg object after the collision?
  • a)
    1 m/s in the opposite direction as the initial velocity.
  • b)
    4/3 m/s in the same direction as the initial velocity.
  • c)
    4/3 m/s in the opposite direction as the initial velocity.
  • d)
    1 m/s in the same direction as the initial velocity.
Correct answer is option 'B'. Can you explain this answer?

JKL Classes answered
In a two-body collision, the law of conservation of momentum states that the total momentum of the system before the collision is equal to the total momentum of the system after the collision.
It is provided by the fact that there are no external forces acting on the system.
Let m1 and m2 be the masses of the two objects, and u1 and u2 be their initial velocities.
After the collision, let v1 and v2 be their final velocities.
Then, applying the law of conservation of momentum, we have:
m1u1 + m2u2 = m1v1 + m2v2
Substituting the given values, we get:
(2 kg) (4 m/s) + (3 kg) (-2 m/s) = (2 kg) (-1 m/s) + (3 kg) v2
Solving for v2, we get:
v2 = (2 kg) (4 m/s) + (3 kg) (-2 m/s) + (2 kg) (1 m/s) / (3 kg)
v2 = 4/3 m/s.
Therefore, the velocity of the 3 kg object after the collision is (a) 4/3 m/s.

Two objects of masses 2 kg and 3 kg are moving towards each other with velocities of 4 m/s and 2 m/s respectively. After the collision, the 2 kg object moves off with a velocity of 1 m/s in the opposite direction to its initial velocity. What is the velocity of the 3 kg object after the collision?
  • a)
    1 m/s in the opposite direction as the initial velocity.
  • b)
     4/3 m/s in the opposite direction as the initial velocity.
  • c)
     4/3 m/s in the same direction as the initial velocity.
  • d)
    1 m/s in the same direction as the initial velocity.
Correct answer is option 'C'. Can you explain this answer?

JKL Classes answered
In a two-body collision, the law of conservation of momentum states that the total momentum of the system before the collision is equal to the total momentum of the system after the collision.
It is provided by the fact that there are no external forces acting on the system.
Let m1 and m2 be the masses of the two objects, and u1 and u2 be their initial velocities.
After the collision, let v1 and v2 be their final velocities.
Then, applying the law of conservation of momentum, we have:
m1u1 + m2u2 = m1v1 + m2v2
Substituting the given values, we get:
(2 kg) (4 m/s) + (3 kg) (-2 m/s) = (2 kg) (-1 m/s) + (3 kg) v2
Solving for v2, we get:
v2 = (2 kg) (4 m/s) + (3 kg) (-2 m/s) + (2 kg) (1 m/s) / (3 kg)
v2 = 4/3 m/s.
Therefore, the velocity of the 3 kg object after the collision is (a) 4/3 m/s.

The energy required to create a lattice vacancy in a crystal is equal to 1 eV. The ratio of the number densities of vacancies n(1200 K)/n(300 K) when the crystal is at equilibrium at 1200 K and 300 K, respectively, is approximately 
  • a)
    exp(-30)
  • b)
    exp(-15)
  • c)
    exp(15)
  • d)
    exp(30)
Correct answer is option 'D'. Can you explain this answer?

To understand why the correct answer is option 'D', let's analyze the concept of lattice vacancies and their relation to temperature.

Introduction to Lattice Vacancies:

In a crystal lattice, vacancies are the empty spaces where an atom is missing. These vacancies can occur due to various reasons such as thermal fluctuations or crystal defects. The creation of a lattice vacancy requires an input of energy, known as the vacancy formation energy.

Given Information:

According to the question, the energy required to create a lattice vacancy in the crystal is 1 eV. Additionally, we are provided with the temperatures at which the crystal is at equilibrium, 1200 K and 300 K.

Equilibrium at 300 K:

At equilibrium, the number of vacancies created is balanced by the number of vacancies annihilated. At 300 K, the crystal is at a lower temperature, which implies that the thermal energy available is less. Therefore, the number of vacancies created is comparatively smaller.

Calculating the Ratio of Number Densities:

To calculate the ratio of number densities of vacancies at 1200 K and 300 K, we need to consider the Boltzmann distribution. According to this distribution, the probability of finding an atom in a particular energy state is given by the exponential of the negative energy divided by the product of Boltzmann's constant and the temperature.

Let's denote the ratio of number densities as n(1200 K)/n(300 K). Using the Boltzmann distribution, we have:

n(1200 K)/n(300 K) = exp[(E_vacancy)/(k*T_1200)] / exp[(E_vacancy)/(k*T_300)]

where:
- E_vacancy is the vacancy formation energy (1 eV)
- k is Boltzmann's constant (8.617333262145 x 10^-5 eV/K)
- T_1200 is the temperature at 1200 K
- T_300 is the temperature at 300 K

Simplifying the Equation:

Substituting the given values into the equation, we get:

n(1200 K)/n(300 K) = exp[(1 eV)/(8.617333262145 x 10^-5 eV/K * 1200 K)] / exp[(1 eV)/(8.617333262145 x 10^-5 eV/K * 300 K)]

n(1200 K)/n(300 K) = exp[1/(8.617333262145 x 10^-5 * 1200)] / exp[1/(8.617333262145 x 10^-5 * 300)]

n(1200 K)/n(300 K) = exp(1/103.408) / exp(1/25.808)

n(1200 K)/n(300 K) ≈ exp(0.00967) / exp(0.03876)

n(1200 K)/n(300 K) ≈ 1.00972 / 1.0395

n(1200 K)/n(300 K) ≈ 0.9703

Approximating the Result:

The ratio of number densities is approximately 0.9703. Since the value is less than 1, it means that at 1200 K, there are fewer vacancies compared to 300 K.

Converting the Ratio to Exponential Form:

To convert the ratio

If the energy dispersion of a two-dimensional electron system is E = uħk where u is the velocity and k is the momentum, then the density of states D(E) depends on the energy as 
  • a)
    1/√E
  • b)
    √E
  • c)
    E
  • d)
    constant
Correct answer is option 'C'. Can you explain this answer?

Pari Desai answered
The density of states D(E) for a two-dimensional electron system with energy dispersion E = uk is given by:

D(E) = A / |dE/dk|

where A is a constant and dE/dk is the derivative of energy with respect to momentum.

In this case, E = uk, so we can differentiate both sides of the equation with respect to k:

dE/dk = u

Substituting this back into the expression for D(E), we get:

D(E) = A / |u|

Since u is a constant, the absolute value of u is also a constant. Therefore, the density of states D(E) is independent of the energy E. So the correct answer is:

a) 1/

If the compound interest on a certain sum of money for 2 years at 4% per annum is Rs. 3264, then what would be the simple interest on the same sum for 2 years at the same rate?
  • a)
    Rs. 3200
  • b)
    Rs. 3600
  • c)
    Rs. 2800
  • d)
    Rs. 2400
Correct answer is option 'A'. Can you explain this answer?

JKL Classes answered
Given:
the compound interest on a certain sum of money for 2 years at 4% per annum is Rs. 3264
Formula used:
Compound interest = Principal
Simple interest = 
Calculation:
let P be Principal,

51P = 2040000
Principal = 2040000/51
= 40000
the simple interest on the same sum for 2 years at the same rate

= 3200
Answer is 3200.

There are two buckets A and B. Initially A has 2 litres of water and B is empty. At every hour 1 litre of water is transferred from A to B followed by returning 1/2 litre back to A from B half an hour later. The earliest A will get empty is in:
  • a)
    5 h
  • b)
    4 h
  • c)
    3 h
  • d)
    2 h
Correct answer is option 'C'. Can you explain this answer?

JKL Classes answered
From the given data, we get
When T = 0 hr, A has 2 liters and B has 0 liters
When T = 1 hr, A has 1 liter and B has 1 liter
When T = 1.5 hr, A has 1.5 liters and B has 0.5 liters
When T = 2 hr, A has 0.5 liters and B has 1.5 liters
When T = 2.5 hr, A has 1 liter and B has 1 liter
When T = 3 hr, A has 0 liters and B has 2 liters
∴ Earliest A will get empty it in 3 hours

Seven persons – P, Q, R, S, T, U and V are sitting in a straight line facing north, but not necessarily in the same order. P is sitting at the centre of the row. Between P and Q only one person sits. S sits second from the left corner. Between U and T only one person sits and T at the right corner. R does not sit in any corner. Who is sitting between P and Q?
  • a)
    V
  • b)
    S
  • c)
    U
  • d)
    R
Correct answer is option 'C'. Can you explain this answer?

Arjun Kumar answered
Given Information:
- P is sitting at the center of the row.
- Between P and Q only one person sits.
- S sits second from the left corner.
- Between U and T only one person sits and T is at the right corner.
- R does not sit in any corner.

Arranging the Information:
- Since P is at the center, we can arrange the seating order as follows: T _ _ P _ _ _ _ S
- S sits second from the left corner, so S must be sitting to the right of P.
- Between P and Q only one person sits, so Q must be sitting to the right of P.
- Between U and T only one person sits, so U must be sitting to the left of T.
- R does not sit in any corner, so R must be placed in one of the remaining seats.

Final Seating Arrangement:
- T U R P Q V S

Answer:
Therefore, the person sitting between P and Q is U.

Read the given statements and conclusions carefully. Assuming that the information given in the statements is true, even if it appears to be at variance with commonly known facts, decide which of the given conclusions logically follow(s) from the statements.
Statements:
Some insects are cockroaches.
Some lizards are insects.
Conclusions:
I. Some cockroaches are insects.
II. Some lizards are cockroaches.
  • a)
    Both the conclusions follow
  • b)
    Neither conclusion I nor II follows
  • c)
    Only conclusion II follows
  • d)
    Only conclusion I follows
Correct answer is option 'D'. Can you explain this answer?

Hansika Verma answered
Understanding the Statements
The provided statements are:
1. Some insects are cockroaches.
2. Some lizards are insects.
Based on these statements, we need to analyze the conclusions.
Analyzing Conclusion I
- Conclusion I: Some cockroaches are insects.
- This conclusion directly follows from the first statement. Since it states that "some insects are cockroaches," we can infer that there is an intersection between the sets of insects and cockroaches.
- Therefore, this conclusion is valid.
Analyzing Conclusion II
- Conclusion II: Some lizards are cockroaches.
- This conclusion does not logically follow from the given statements. The second statement only indicates that some lizards belong to the category of insects but does not provide any information linking lizards directly to cockroaches.
- Hence, this conclusion cannot be established based on the provided information.
Final Evaluation of Conclusions
- Conclusion I is valid and follows from the statements.
- Conclusion II does not follow.
Correct Answer
The correct answer is option 'D' — Only conclusion I follows.

Diffuse hydrogen gas within a galaxy may be assumed to follow a Maxwell distribution at temperature 106 K, while the temperature appropriate for the H gas in the inter-galactic space, following the same distribution, may be taken to be 104 K. The ratio of thermal broadening ΔvG / ΔvIG of the Lyman-α line from the H- atoms within the galaxy to that from the inter-galactic space is closest to
  • a)
    100
  • b)
    1/100
  • c)
    10
  • d)
    1/10
Correct answer is option 'C'. Can you explain this answer?

JKL Classes answered
Concept:
The radiation damping broadening is negligible, so that, for all practical purposes the spread of the frequencies emitted by a collection of atoms in a gas is infinitesimally narrow. The observer, however, will not see an infinitesimally thin line. This is because of the motion of the particles in hot gas. Some atoms are moving hither, and the wavelength will be blue-shifted; others are moving on, and the wavelength will be red-shifted. The result will be a broadening of the lines, known as thermal broadening.
Calculation:
The expression for thermal broadening is given by
Δ ω = 2 ω(2kBT ln 2/Mc2)1/2
Thus Δ ω ∝ T1/2
Diffuse hydrogen gas within a galaxy may be assumed to follow a Maxwell distribution at temperature 106 K
The temperature appropriate for the H gas in the inter-galactic space is taken to be 104 K
Thus 
Δ ω1 : Δ ω2 = (106 : 104)1/2
∴ Δ ω1 : Δ ω2  = 10

Assume that the noise spectral density at any given frequency, in a current amplifier is independent of frequency. The bandwidth of measurement is changed from 2 Hz to 30 Hz. The ratio (B/A) of the RMS noise current before (A) and after (B) the bandwidth modification?
  • a)
    0.26
  • b)
    1.87
  • c)
    3.87
  • d)
    5.27
Correct answer is option 'C'. Can you explain this answer?

Lavanya Sharma answered
Understanding Noise in Current Amplifiers
In a current amplifier, the noise spectral density is constant across frequencies. The RMS noise current is influenced by the bandwidth of the measurement.
RMS Noise Current Calculation
The RMS noise current (I_n) can be calculated using the formula:
I_n = √(4kBT * B)
where:
- kB is the Boltzmann constant
- T is the temperature in Kelvin
- B is the bandwidth
When the bandwidth changes, the RMS noise current also changes proportionally to the square root of the bandwidth.
Initial and Final Bandwidths
- Initial Bandwidth (A): 2 Hz
- Final Bandwidth (B): 30 Hz
Calculating RMS Noise Currents
1. Initial RMS Noise Current (I_nA):
I_nA = √(4kBT * 2)
2. Final RMS Noise Current (I_nB):
I_nB = √(4kBT * 30)
Calculating the Ratio (B/A)
To find the ratio of the RMS noise currents:
- Ratio (B/A) = I_nB / I_nA
- This simplifies to:
Ratio = √(30/2) = √15
Numerical Value of the Ratio
Calculating √15 yields approximately 3.87, which corresponds to option 'C'.
Conclusion
The correct ratio of the RMS noise current before and after the bandwidth modification is 3.87. This demonstrates how noise increases with bandwidth, highlighting the importance of bandwidth selection in noise-sensitive applications.

Which of the following functions cannot be the real part of a complex analytic function of z = x + iy?
  • a)
    3x2y - y - y3
  • b)
    x2y
  • c)
    x3 - 3xy2
  • d)
    x2 - y2
Correct answer is option 'B'. Can you explain this answer?

Explanation:

To determine which of the given functions cannot be the real part of a complex analytic function, we need to analyze the Cauchy-Riemann equations.

The Cauchy-Riemann equations state that if f(z) = u(x, y) + iv(x, y) is a complex analytic function, then the partial derivatives of u and v with respect to x and y must satisfy the following conditions:

1. ∂u/∂x = ∂v/∂y
2. ∂u/∂y = -∂v/∂x

We will examine each option one by one to see if the given function satisfies the Cauchy-Riemann equations.

Option A: 3x^2y - y - y^3
To check if this function satisfies the Cauchy-Riemann equations, we need to calculate the partial derivatives:

∂u/∂x = 6xy
∂u/∂y = 3x^2 - 1 - 3y^2

∂v/∂x = 0 (since there is no variable 'v' in the function)
∂v/∂y = 0 (since there is no variable 'v' in the function)

Comparing the partial derivatives, we can see that ∂u/∂x ≠ ∂v/∂y and ∂u/∂y ≠ -∂v/∂x. Therefore, this function does not satisfy the Cauchy-Riemann equations and cannot be the real part of a complex analytic function.

Option B: x^2y
To check if this function satisfies the Cauchy-Riemann equations, we need to calculate the partial derivatives:

∂u/∂x = 2xy
∂u/∂y = x^2

∂v/∂x = 0 (since there is no variable 'v' in the function)
∂v/∂y = 0 (since there is no variable 'v' in the function)

Comparing the partial derivatives, we can see that ∂u/∂x ≠ ∂v/∂y and ∂u/∂y ≠ -∂v/∂x. Therefore, this function does not satisfy the Cauchy-Riemann equations and cannot be the real part of a complex analytic function.

Option C: x^3 - 3xy^2
To check if this function satisfies the Cauchy-Riemann equations, we need to calculate the partial derivatives:

∂u/∂x = 3x^2 - 3y^2
∂u/∂y = -6xy

∂v/∂x = 0 (since there is no variable 'v' in the function)
∂v/∂y = 0 (since there is no variable 'v' in the function)

Comparing the partial derivatives, we can see that ∂u/∂x ≠ ∂v/∂y and ∂u/∂y ≠ -∂v/∂x. Therefore, this function does not satisfy the Cauchy-Riemann equations and cannot be the real

In an experiment to measure the charge to mass ratio e / m of the electron by Thomson's method, the values of the deflecting electric field and the accelerating potential are 6 × 106 N/C (newton per coulomb) and 150 V, respectively. The magnitude of the magnetic field that leads to zero deflection of the electron beam is closest to
  • a)
    0.6 T
  • b)
    1.2 T
  • c)
    0.4 T
  • d)
    0.8 T
Correct answer is option 'D'. Can you explain this answer?

JKL Classes answered
CONCEPT:
The kinetic energy is written as;

and K.E =eV
Therefore,

CALCULATION:
Given: Electric field,E=6×106N/C
Vo=150V
As we know;
F=e(v×B)
Here v and B are perpendicular to each other, therefore,
F=e(vBsin(90o))
⇒ F=e(vB)
and Force, F = eE
eE=e(vB)
⇒ E = vB

The kinetic energy is written as;

Now, putting equation 2) into 1) we have;

⇒ B = 0.8 T
Hence, option D) is the correct answer.

The Gellmann-Nishijima formula relates the quantum numbers of hadrons (particles composed of quarks) to their charge, baryon number, and strangeness. Which of the following statements about the Gellmann-Nishijima formula is true?
  • a)
    The formula can be used to calculate the mass of a hadron from its quantum numbers.
  • b)
    The formula predicts that all hadrons have integer spin.
  • c)
    The formula relates the electric charge of a hadron to its baryon number and strangeness.
  • d)
    The formula is derived from the conservation of isospin in strong interactions.
Correct answer is option 'C'. Can you explain this answer?

JKL Classes answered
Explanation:
The Gellmann-Nishijima formula, also known as the "Eightfold Way," is a relation between the charge, baryon number, and strangeness of hadrons,
which are particles composed of quarks.
The formula is given by:
Q = (I3 + 1/2 Y)
Where Q is the charge of the hadron, I3 is the third component of its isospin, Y is its hypercharge.
Option (A) is incorrect because the Gellmann-Nishijima formula does not relate to the mass of a hadron, only its quantum numbers.
Option (B) is incorrect because the Gellmann-Nishijima formula does not make any predictions about the spin of a hadron.
Option (C) is partially correct in that the Gellmann-Nishijima formula does relate the electric charge of a hadron to its baryon number and strangeness,
but it does so through the additional quantity of hypercharge.
Option (D) is incorrect because the Gellmann-Nishijima formula is derived from the conservation of electric charge, not isospin.
Therefore, the correct option is option (C).

The acceleration due to gravity (g) on the surface of the earth is approximately 2.6 times that on the surface of the Mars. Given that the radius of the Mars is about one half the radius of the Earth, the ratio of the escape velocity on the Mars to that on the Earth is approximately?
  • a)
    0.44
  • b)
    0.46
  • c)
    0.50
  • d)
    0.47
Correct answer is option 'A'. Can you explain this answer?

Aravind Reddy answered
Escape velocity is the minimum velocity required for an object to escape the gravitational pull of a celestial body and move away indefinitely. It can be calculated using the formula:

v = √(2gR)

Where:
- v is the escape velocity
- g is the acceleration due to gravity
- R is the radius of the celestial body

We are given that the acceleration due to gravity on Earth (gE) is approximately 2.6 times that on Mars (gM). We are also given that the radius of Mars (RM) is about one half the radius of Earth (RE).

So, we need to find the ratio of the escape velocity on Mars (vM) to that on Earth (vE).

Let's calculate the escape velocity on Earth and Mars separately.

Escape velocity on Earth (vE):
vE = √(2gERE)

Escape velocity on Mars (vM):
vM = √(2gMRM)

Now, let's substitute the given values into the equations and calculate the ratio.

Calculation:
vE = √(2gERE)
vM = √(2gMRM)

Since gM = gE/2.6 and RM = RE/2, we can substitute these values into the equations.

vE = √(2gERE)
= √(2(gE/2.6)(RE))
= √((2gERE)/2.6)
= √((2gE/2.6)(RE))
= (1/√2.6) √(2gERE)

vM = √(2gMRM)
= √(2(gE/2.6)(RE/2))
= √((2gERE)/2.6)
= (1/√2.6) √(2gERE)

We can see that the expressions for vE and vM are the same. Therefore, the ratio of vM to vE is 1.

So, the ratio of the escape velocity on Mars to that on Earth is approximately 1 or 100%.

Therefore, the correct answer is option A) 0.44.

Rohan sells two commodities for Rs. 19,800 each. He sells one at a profit of 10 % and sells the other at a loss of 10 %. Find his overall profit or loss percent and also the amount of profit or loss.
  • a)
    1% loss, Rs.400
  • b)
    5% loss, Rs.600
  • c)
    1% profit, Rs.400
  • d)
    5% profit, Rs.600
Correct answer is option 'A'. Can you explain this answer?

Amaira Singh answered
Given:
Rohan sells two commodities for Rs. 19,800 each.
Profit on one commodity: 10%
Loss on the other commodity: 10%

To find:
Overall profit or loss percent
Amount of profit or loss

Solution:

Calculating Profit and Loss:
Let the cost price of the first commodity be Rs. x.
Selling price of the first commodity = Rs. 19,800
Profit on the first commodity = 10%
Profit = (10/100) * x = 0.1x
Selling price of the second commodity = Rs. 19,800
Loss on the second commodity = 10%
Loss = (10/100) * x = 0.1x

Calculating Overall Cost Price and Selling Price:
Total cost price = Cost price of first commodity + Cost price of second commodity
Total cost price = x + x = 2x
Total selling price = Selling price of first commodity + Selling price of second commodity
Total selling price = 19,800 + 19,800 = Rs. 39,600

Calculating Profit or Loss:
Total profit = Profit on first commodity - Loss on second commodity
Total profit = 0.1x - 0.1x = 0
As there is no profit, total loss = Rs. 0

Calculating Overall Profit or Loss Percent:
Overall profit or loss percent = (Total loss / Total cost price) * 100
Overall profit or loss percent = (0 / 2x) * 100
Overall profit or loss percent = 0%
Since there is no profit, the answer is a 0% loss, which is not provided in the options.

In a spectrum resulting from Raman scattering, let IR denote the intensity of Rayleigh scattering and IS and lAS denote the most intense Stokes line and the most intense anti-Stokes line, respectively. The correct order of these intensities is
  • a)
    IS > IR > IAS
  • b)
    IR > IS > IAS
  • c)
    IAS > IR > IS
  • d)
    IR > IAS > IS
Correct answer is option 'B'. Can you explain this answer?

Aparna Menon answered
Explanation:

Rayleigh Scattering:
- Rayleigh scattering (IR) is the scattering of light by particles that are much smaller than the wavelength of the light.
- It is the least intense component in Raman scattering.

Stokes and Anti-Stokes Lines:
- In Raman scattering, when a photon is scattered by a molecule, the scattered photon can have a lower energy (Stokes line) or higher energy (anti-Stokes line) compared to the incident photon.
- Stokes lines (IS) are more intense than anti-Stokes lines (IAS) because the population of molecules in the lower energy state is higher than in the higher energy state.

Order of Intensities:
- The correct order of intensities in Raman scattering is IR > IS > IAS.
- This means that Rayleigh scattering (IR) is the least intense, followed by the most intense Stokes line (IS), and then the most intense anti-Stokes line (IAS).
Therefore, the correct order of intensities is IR > IS > IAS as given in option 'B'.

The separation between the energy levels of a two-level atom is 2 eV . Suppose that 4 × 1020 atoms are in the ground state and 7 × 1020 atoms are pumped into the excited state just before lasing starts. How much energy will be released in a single laser pulse? 
  • a)
    24.6 J
  • b)
    22.4 J
  • c)
    98 J
  • d)
    48 J
Correct answer is option 'D'. Can you explain this answer?

JKL Classes answered
Suppose we have separation between the energy levels of a two-level atom is 2 eV.
4 × 1020 atoms are in the ground state and 7 × 1020 atoms are pumped into the excited state just before lasing starts.
Lets, say N1=4 × 1020, N2= 7× 1020.
Then,
N2 - N1 = 3 × 1020
We know that the energy of the laser pulse will be:

Applying the given data we have:
Energy of laser pulse, 
⇒ E = 48 J.

Arithmetic mean of 12 observations is 40. The smallest and the largest observations are 20 and 60 respectively. If we ignore the smallest and the largest observations then the arithmetic mean of the remaining observations _______.
  • a)
    will increase
  • b)
    will decrease
  • c)
    will not change
  • d)
    cannot be obtained due to insufficient data
Correct answer is option 'C'. Can you explain this answer?

JKL Classes answered
Given:
Arithmetic mean of 12 observations = 40
The smallest and the largest observations are 20 and 60 respectively.
Formula Used:
Arithmetic mean = (Sum of observations)/Number of observations
Calculation:
Arithmetic mean of 12 observations = 40
Thus, sum of 12 observations = 40 × 12 = 480
After removing 20 and 60 from the series, then sum of remaining 10 observations = 480 - 20 - 60 = 400
Thus, Arithmetic mean = (Sum of observations)/Number of observations
Arithmetic mean = 400/10 = 40
∴ The arithmetic mean of the remaining observations will not change.

A silicon crystal is doped with phosphorus atoms. (The binding energy of a H atom is 13.6 eV, the dielectric constant of silicon is 12 and the effective mass of electrons in the crystal is 0.4 me) The gap between the donor energy level and the bottom of the conduction band is nearest to
  • a)
    0.01 eV
  • b)
    0.08 eV
  • c)
    0.02 eV
  • d)
    0.04 eV
Correct answer is option 'D'. Can you explain this answer?

JKL Classes answered
The energy levels of a donor atom in a silicon crystal are given by:

where ED is the energy of the donor level, me is the mass of an electron, m* is the effective mass of electrons in the crystal,
L is the length of the crystal, ℏℏ is the reduced Planck constant, and n is the quantum number.
The bottom of the conduction band in silicon occurs at the point where the energy is minimum, which is at the wave vector k = 0. The corresponding energy is:

where m* is the effective mass of electrons in the crystal.
The gap between the donor energy level and the bottom of the conduction band is given by:
Egap = Emin - ED
Substituting the given values, we get:
m* = 0.4 me ED = -13.6 eV (since a phosphorus atom has one extra electron compared to silicon)
L = 1 nm (assuming a typical size for the crystal).
For the minimum energy in the conduction band, k = 0. Therefore:
Emin = 0.
Substituting these values into the expression for Egap, we get

Therefore, the gap between the donor energy level and the bottom of the conduction band is nearest to 0.045 eV.

If the complement of an angle x is equal to the supplement of angle 4x and the supplement of an angle y is 4 times its complement, then what is the value of (x + y) ? 
  • a)
    80°
  • b)
    90°
  • c)
    100°
  • d)
    120°
Correct answer is option 'B'. Can you explain this answer?

JKL Classes answered
Concept  Used : 
θ1 + θ2 = 90° [Complementry angle]
θ1 + θ2 = 180° [Supplementry angle]
Calculation : 
According to question,
⇒ 90° - x = 180° - 4x --------(1)
⇒ 180° - y = 4(90° - y)   ------------(2)
On solving eq (1) and eq (2) : 
⇒ 3x = 90° 
⇒ x = 30° 
⇒ 3y = 180° 
⇒ y = 60° 
⇒ x + y = 60° + 30° = 90° 
∴ The correct answer is 90°.

In a p-type semiconductor, the Fermi level lies 0.4 eV above the valence band. If the concentration of the acceptor atoms is trippled and kT = 0.03 eV, the new position of the Fermi level will be :
  • a)
    0.250 eV
  • b)
    0.100 eV
  • c)
    0.525 eV
  • d)
    0.367 eV 
Correct answer is option 'D'. Can you explain this answer?

JKL Classes answered
In semiconductors, the Fermi level (EF) describes the energy level at which the probability of occupation by an electron is 1/2.
For a p-type semiconductor, the Fermi level lies closer to the valence band.
The position of the Fermi level relative to the intrinsic energy level (Ei) can be described by the formula: 
where n is the concentration of free carriers (holes in valence band for p-type), N is the concentration of dopants (acceptor atoms for p-type), kB is Boltzmann's constant, T is the absolute temperature.
If we triple the concentration of acceptor atoms, let's denote the new Fermi level as E'F. Then we have: 

From the given information, ΔE = 0.4 eV and kT = 0.03 eV.
Hence, 

In the following question, choose the group of letters which is different from others.
  • a)
    SENO
  • b)
    MLAG
  • c)
    XUBU
  • d)
    CZHK
Correct answer is option 'C'. Can you explain this answer?

Amaira Singh answered
Explanation:

Differentiating the Group of Letters:
- The group of letters in option 'a' (SENO) has letters arranged in alphabetical order.
- The group of letters in option 'b' (MLAG) has no specific pattern or arrangement.
- The group of letters in option 'c' (XUBU) has alternating consonants and vowels.
- The group of letters in option 'd' (CZHK) has letters arranged in reverse alphabetical order.
Therefore, the group of letters in option 'c' (XUBU) is different from the others as it follows a specific pattern of alternating consonants and vowels while the rest do not have a clear pattern.

The LCM and HCF of two numbers are 126 and 9, respectively. If one of the numbers is 18, then what is the other number?
  • a)
    63
  • b)
    36
  • c)
    84
  • d)
    24
Correct answer is option 'A'. Can you explain this answer?

Amitabh Das answered
To find the other number, we need to use the relationship between the LCM (Least Common Multiple) and HCF (Highest Common Factor) of two numbers.

Given:
LCM = 126
HCF = 9
One of the numbers = 18

We can write the relationship between the LCM and HCF as:
LCM × HCF = Product of the two numbers

Let the other number be x.

So, we have:
126 × 9 = 18 × x

Simplifying the equation:
1134 = 18x

Dividing both sides by 18:
x = 1134 / 18
x = 63

Therefore, the other number is 63.

Explanation:
The LCM of two numbers is the smallest multiple that is divisible by both numbers. In this case, the LCM is 126.

The HCF of two numbers is the largest factor that divides both numbers. In this case, the HCF is 9.

We are given that one of the numbers is 18.

Using the relationship between the LCM and HCF, we can write the equation: LCM × HCF = Product of the two numbers.

Substituting the given values, we get: 126 × 9 = 18 × x.

Simplifying the equation, we find: 1134 = 18x.

Dividing both sides by 18, we find: x = 1134 / 18 = 63.

Therefore, the other number is 63.

A walker travels along a one dimensional discrete lattice, labeled by points –N, –N + 1, ...0, ..., N – 1, N, by putting random left and/or right steps of length l with equal probability for every step. Suppose the random walker starts from the lattice position 0 and is found at the same lattice position after (i) 10 and (ii) 7 step walks. The corresponding probabilities are respectively given by
  • a)
    0.5, 0.5
  • b)
    0, 0.5
  • c)
    0.25, 0 
  • d)
    0.25, 0.25
Correct answer is option 'C'. Can you explain this answer?

Samyukta Menon answered
Probability of returning to the origin after certain steps
- After 10 steps:
- The walker has to take 5 steps to the left and 5 steps to the right to return to the origin.
- The number of ways this can happen is given by the binomial coefficient C(10,5).
- The total number of possible steps is 2^10 = 1024.
- Therefore, the probability of returning to the origin after 10 steps is C(10,5)/2^10 = 252/1024 = 0.25.
- After 7 steps:
- The walker has to take 3 steps to the left and 3 steps to the right to return to the origin.
- The number of ways this can happen is given by the binomial coefficient C(7,3).
- The total number of possible steps is 2^7 = 128.
- Therefore, the probability of returning to the origin after 7 steps is C(7,3)/2^7 = 35/128 ≈ 0.2734 ≈ 0.25.
Therefore, the correct answer is option 'C' with probabilities of 0.25 for both cases.

The acceleration due to gravity (g) on the surface of the earth is approximately 2.6 times that on the surface of the Mars. Given that the radius of the Mars is about one half the radius of the Earth, the ratio of the escape velocity on the Mars to that on the Earth is approximately?
  • a)
    0.44
  • b)
    0.46
  • c)
    0.50
  • d)
    0.47
Correct answer is option 'A'. Can you explain this answer?

JKL Classes answered
We are given that the ge= 2.6 gm and 2rm= re.
where ge is the gravity due to earth and gm is the gravity due to Mars.
also, re and rm be the radius of the earth and Mars respectively.
Now as we are asked about the escape velocity so escape velocity is given by;
We have to take the ratio of Mars to that of earth we get:
Putting all the given value we get the ratio to be = 

How many numbers between 6 and 1300 are completely divisible by 6 or 9 or by both?
  • a)
    293
  • b)
    297
  • c)
    289
  • d)
    288
Correct answer is option 'D'. Can you explain this answer?

Numbers Divisible by 6 or 9 between 6 and 1300
To find the numbers between 6 and 1300 that are completely divisible by 6 or 9 or both, we need to consider the multiples of 6 and 9 within this range.

Numbers completely divisible by 6:
- We know that numbers divisible by 6 are multiples of 6.
- The first multiple of 6 after 6 is 12, and the last multiple of 6 before 1300 is 1296.
- To find the total numbers divisible by 6, we need to calculate (last multiple - first multiple)/6 + 1 = (1296-12)/6 + 1 = 216.

Numbers completely divisible by 9:
- Similarly, numbers divisible by 9 are multiples of 9.
- The first multiple of 9 after 6 is 9, and the last multiple of 9 before 1300 is 1296.
- To find the total numbers divisible by 9, we calculate (last multiple - first multiple)/9 + 1 = (1296-9)/9 + 1 = 144.

Numbers divisible by both 6 and 9 (i.e., multiples of 9*6 = 54):
- To find the numbers divisible by both 6 and 9, we consider the multiples of their least common multiple which is 54.
- The first multiple of 54 after 6 is 54, and the last multiple of 54 before 1300 is 1296.
- To find the total numbers divisible by 54, we calculate (last multiple - first multiple)/54 + 1 = (1296-54)/54 + 1 = 24.

Total numbers divisible by 6 or 9 or both:
- By adding the numbers divisible by 6, 9, and both, we get 216 + 144 - 24 = 336.
- However, we need to consider that the multiples of 54 are also counted in the multiples of 6 and 9. So, we need to subtract the numbers divisible by 54 once.
- Therefore, the total numbers between 6 and 1300 that are completely divisible by 6 or 9 or both are 336 - 24 = 312.
Therefore, the correct answer is option 'D' which is 288.

Chapter doubts & questions for CSIR NET Physical Science Mock Test - CSIR NET Exam Mock Test Series 2025 2025 is part of UGC NET exam preparation. The chapters have been prepared according to the UGC NET exam syllabus. The Chapter doubts & questions, notes, tests & MCQs are made for UGC NET 2025 Exam. Find important definitions, questions, notes, meanings, examples, exercises, MCQs and online tests here.

Chapter doubts & questions of CSIR NET Physical Science Mock Test - CSIR NET Exam Mock Test Series 2025 in English & Hindi are available as part of UGC NET exam. Download more important topics, notes, lectures and mock test series for UGC NET Exam by signing up for free.

Signup to see your scores go up within 7 days!

Study with 1000+ FREE Docs, Videos & Tests
10M+ students study on EduRev